반응형

[고전역학] 6. 진동(2: 감쇠 조화운동, 강제 조화진동)



여기서는 공기 저항으로 인한 마찰을 고려할 것이다. 다음 그림의 감쇠 조화 진동자에서

평형점으로부터의 변위를 \(x\)라고 하면, 물체가 받는 복원력이 \(-kx\), 저항으로 인한 마찰력(retarding force)은 \(-c\dot{x}\)(\(c\)는 비례상수)이므로 이 물체의 운동방정식은 \(m\ddot{x}=-kx-c\dot{x}\)이고$$m\ddot{x}+c\dot{x}+kx=0$$이다. 이 운동방정식을$$\ddot{x}+\frac{c}{m}\dot{x}+\frac{k}{m}x=0$$으로 나타낼 수 있고, 감쇠인자(damping factor) \(\gamma\)를$$\gamma=\frac{c}{2m}$$으로 정의하고, 각진동수 \(\displaystyle\omega_{0}^{2}=\frac{k}{m}\)을 이용하여 이 운동방정식을$$\ddot{x}+2\gamma\dot{x}+\omega_{0}^{2}x=0$$이다. \(q=\sqrt{\gamma^{2}-\omega_{0}^{2}}\)라고 하면, \(\gamma>q\)이고 이 운동방정식의 해는$$x(t)=A_{1}e^{-(\gamma-q)t}+A_{2}e^{-(\gamma+q)t}$$이다.

\(q\)가 실수이고 \(q>0\)이면, 과다감쇠(overdamping)이고, \(x(t)=A_{1}e^{-(\gamma-q)t}+A_{2}e^{-(\gamma+q)t}\)이다.

\(q\)가 실수이고 \(q=0\)이면, 임계감쇠(critical damping)이고, \(x(t)=(At+B)e^{-\gamma t}\)이다.

\(q\)가 허수이면(\(\gamma^{2}-\omega_{0}^{2}<0\)), 미급감쇠(underdamping)이고,$$\omega_{d}=\sqrt{\omega_{0}^{2}-\gamma^{2}}=\sqrt{\frac{k}{m}-\frac{c^{2}}{4m^{2}}}$$로 놓고, 오일러 항등식 \(e^{i\omega t}=\cos\omega t+i\sin\omega t\)를 이용하여$$x(t)=e^{-\gamma t}A\cos(\omega_{d}t+\theta_{0})$$또는$$x(t)=e^{-\gamma t}A\sin(\omega_{d}t+\phi_{0})$$으로 나타낼 수 있으며, 주기는$$T_{d}=\frac{2\pi}{\omega_{d}}=\frac{2\pi}{\sqrt{\omega_{0}^{2}-\gamma^{2}}}$$이다.

(과다감쇠, 임계감쇠, 미급감쇠 조화 진동자의 그래프) 


감쇠 조화 진동자에서 총 에너지는 운동에너지와 위치에너지의 합$$E=\frac{1}{2}m\dot{x}^{2}+\frac{1}{2}kx^{2}$$이고,$$\frac{dE}{dt}=m\dot{x}\ddot{x}+kx\dot{x}=(m\ddot{x}+kx)\dot{x}$$이며 \(m\ddot{x}+c\dot{x}+kx=0\)이므로 \(m\ddot{x}+kx=-c\dot{x}\)이고 따라서$$\frac{dE}{dt}=-c\dot{x}^{2}$$이다. 이것은 총 에너지가 시간에 따라 감소해서 역학적 에너지가 마찰열의 형태로 소실됨을 나타낸다.

미급감쇠인 조화진동자의 에너지 손실률은 진동자의 Q인자(quality factor)라는 \(Q\)값으로 나타낼 수 있다. Q인자는 한 진동주기 \(T_{d}\)동안 진동자의 전체 에너지를 손실된 에너지로 나눈 다음, \(2\pi\)를 곱한 값으로 정의된다.$$\frac{dE}{dt}=-c\dot{x}^{2}$$이므로 \(\dot{x}\)를 계산해야 한다.$$x(t)=Ae^{-\gamma t}\sin(\omega_{d}t+\phi_{0})$$이고,$$\dot{x}=-Ae^{\gamma t}(\gamma\sin(\omega_{d}t+\phi_{0})-\omega_{d}\cos(\omega_{d}t+\phi_{0}))$$이므로 한 주기 \(\displaystyle T_{d}=\frac{2\pi}{\omega_{d}}\)동안 진동자가 잃은 에너지는$$\Delta E=\int_{0}^{T_{d}}{\frac{dE}{dt}dt}$$이다. \(\theta=\omega_{d}t+\phi_{0}\)이라고 하면$$\begin{align*}\Delta E&=\frac{1}{\omega_{d}}\int_{0}^{2\pi}{\frac{dE}{dt}d\theta}=\frac{1}{\omega_{d}}\int_{0}^{2\pi}{e^{-2\gamma t}(\gamma^{2}\sin^{2}\theta-2\gamma\omega_{d}\sin\theta\cos\theta+\omega_{d}^{2}\cos^{2}\theta)d\theta}\end{align*}$$이고, 한 주기 동안 \(e^{-2\gamma t}\)의 값이 크게 변하지 않아 상수취급 할 수 있기 때문에 바깥으로 빼낼 수 있다. 이 적분을 계산하면$$\begin{align*}\Delta E&=-\frac{cA^{2}}{\omega_{d}}e^{-2\gamma t}\int_{0}^{2\pi}{(\gamma^{2}\sin^{2}\theta-2\gamma\omega_{d}\sin\theta\cos\theta+\omega_{d}^{2}\cos^{2}\theta)d\theta}\\&=-\frac{cA^{2}}{\omega_{d}}\pi e^{-2\gamma t}(\gamma^{2}+\omega_{d}^{2})\\&=-cA^{2}e^{-2\gamma t}\omega_{0}^{2}\left(\frac{\pi}{\omega_{d}}\right)\\&=-\gamma m\omega_{0}^{2}A^{2}e^{-2\gamma t}T_{d}\\&=\left(\frac{1}{2}mA^{2}\omega_{0}^{2}e^{-\frac{t}{\tau}}\right)\frac{T_{d}}{\tau}\,\left(\gamma=\frac{1}{2\tau}\right)\end{align*}$$이므로$$\frac{\Delta E}{E}=\frac{T_{d}}{\tau},\,E=\frac{1}{2}m\omega_{0}^{2}A^{2}e^{-\frac{t}{\tau}}$$이다. 따라서 Q인자 \(Q\)는 다음과 같다.$$Q=\frac{2\pi}{\frac{T_{d}}{\tau}}=\frac{2\pi\tau}{\frac{2\pi}{\omega_{d}}}=\omega_{d}\tau=\frac{\omega_{d}}{2\gamma}$$


어떤 조화진동을 하는 물체에 \(F_{0}\cos\omega t\)의 구동력(driving force)을 가해주면 이 물체의 운동방정식은 다음과 같다.$$m\ddot{x}=-kx-c\dot{x}+F_{0}\cos\omega t$$구동력의 진동수 \(\omega\)가 물체의 고유진동수 \(\omega\)와 같게 되면, 공명(resonance)현상이 일어난다(그네를 밀 때, 적당히 때를 맞추어서 밀면 진동 폭이 상당히 커진다). 

감쇠가 없는 조화진동을 하는 물체는 고유 진동수 \(\displaystyle\omega_{0}=\sqrt{\frac{k}{m}}\)로 진동한다. 실제로는 저항(마찰)이 존재해서 시간이 지나면 운동을 멈추게 된다. 구동력을 받는 조화진동을 하는 물체는 진동수가 \(\omega_{d}\)인 진동을 하다가 시간이 지나면 구동 진동수 \(\omega\)로 진동하게 된다. 이것은 두 진동수가 중첩되었다가 하나는 소멸하고, 다른 하나는 지속된다. 소멸하는 운동을 과도상태(transient state), 구동 진동수로 계속되는 운동을 정상상태(steady state)라고 한다. 과도상태는 동차해를 갖고, 정상상태는 비동차해를 갖는다.

이 운동방정식의 해를 구하기 위해 구동력을 \(F=F_{0}e^{i\omega t}\), 정상상태의 해로 \(x(t)=Ae^{i(\omega t-\phi)}\)라고 하면$$m\frac{d^{2}}{dt^{2}}(Ae^{i(\omega t-\phi)})+c\frac{d}{dt}(Ae^{i(\omega t-\phi)})+kAe^{i(\omega t-\phi)}=F_{0}e^{i\omega t}$$이므로$$-m\omega^{2}A+i\omega cA+kA=F_{0}e^{i\phi}=F_{0}(\cos\phi+i\sin\phi)$$이고 실수부와 허수부를 분리하면$$\begin{align*}A(k-m\omega^{2})&=F_{0}\cos\phi\\c\omega A&=F_{0}\sin\phi\end{align*}$$이다. 이때$$\tan\phi=\frac{\sin\phi}{\cos\phi}=\frac{c\omega}{k-m\omega^{2}}$$이고$$A^{2}(k-m\omega^{2})^{2}+c^{2}\omega^{2}A^{2}=F_{0}^{2}$$이므로 정상상태진동의 진폭 \(A(\omega)\)는$$A(\omega)=\frac{F_{0}}{\sqrt{(k-m\omega^{2})^{2}+c^{2}\omega^{2}}}$$이때 \(\displaystyle\omega_{0}^{2}=\frac{k}{m},\,\gamma=\frac{c}{2m}\)이므로$$\begin{align*}\tan\phi&=\frac{2\gamma\omega}{\omega_{0}^{2}-\omega^{2}}\\A(\omega)&=\frac{F_{0}}{m\sqrt{(\omega_{0}^{2}-\omega^{2})^{2}+4\gamma^{2}\omega^{2}}}\end{align*}$$이고 \(\displaystyle\frac{dA}{d\omega}=0\)이 되게 하는 \(\omega\)를 공명진동수라고 하고, \(\omega_{r}\)로 나타낸다. 이때$$\omega_{r}^{2}=\omega_{0}^{2}-2\gamma^{2}$$가 성립한다. 강제 진동이 없으면 \(\omega_{d}=\sqrt{\omega_{0}^{2}-\gamma^{2}}\)이므로$$\omega_{r}^{2}=\omega_{d}^{2}-\gamma^{2}$$이다. \(\displaystyle\gamma>\frac{\omega_{0}}{\sqrt{2}}\)이면, 이 때의 진폭이 \(\omega\)의 감소함수가 되기 때문에 공명이 일어나지 않는다. \(\displaystyle\gamma^{2}=\frac{\omega_{0}^{2}}{2}\)일 때를 고려하면$$A(\omega)=\frac{F_{0}}{m\sqrt{(\omega_{0}^{2}-\omega^{2})^{2}+2\omega_{0}^{2}\omega^{2}}}=\frac{F_{0}}{m\sqrt{\omega_{0}^{4}+\omega^{4}}}$$이므로 \(\omega\)의 감소함수이다.

(진폭과 위상차의 그래프)


공명 상태에서의 정상상태 해가 갖는 진폭 \(A_{\max}\)는$$A_{\max}=\frac{F_{0}}{2m\gamma\sqrt{\omega_{0}^{2}-\gamma^{2}}}$$이고, 감쇠가 약한 경우는$$A_{\max}\simeq\frac{F_{0}}{2m\gamma\omega_{0}}$$으로 나타낼 수 있다.

(기계가 떨리는 진동의 전달을 최소화하기 위해 고무와 용수철을 사용한다.)


감쇠가 약한 경우, \(\gamma\ll\omega_{0}\)이므로$$\begin{align*}\omega_{0}^{2}-\omega^{2}&=(\omega_{0}+\omega)(\omega_{0}-\omega)\\&\simeq2\omega_{0}(\omega_{0}-\omega)\\4\gamma^{2}\omega^{2}&\simeq4\gamma^{2}\omega_{0}^{2}\end{align*}$$이고$$A(\omega)=\frac{A_{\max}\gamma}{\sqrt{(\omega_{0}-\omega)^{2}+\gamma^{2}}}$$이다. \(|\omega_{0}-\omega|=\gamma\)이면, \(\displaystyle A^{2}=\frac{1}{2}A_{\max}^{2}\)이다. 

앞에서 언급했던 Q인자를 여기에서는 강제 진동자의 에너지 손실률로 볼 수 있고, 감쇠가 약한 경우$$Q=\frac{\omega_{d}}{2\gamma}\approx\frac{\omega_{0}}{2\gamma}$$이다. 그러므로$$\Delta\omega=2\gamma\approx\frac{\omega_{0}}{Q}$$이고 \(\omega=2\pi f\)이므로$$\frac{\Delta\omega}{\omega_{0}}=\frac{\Delta f}{f_{0}}\simeq\frac{1}{Q}$$이다. 여기서 \(\Delta\omega\)는 최대 진폭의 \(\displaystyle\frac{1}{\sqrt{2}}=0.707\)배인 진폭의 곡선 상의 점들 사이의 진동수 간격이다. 


*전기회로와 역학계의 비교

 

역학계 

 

전기계 

\(x\) 

변위 

\(q\) 

전하량 

\(\dot{x}\) 

속도 

\(\dot{q}=i\) 

전류 

\(m\) 

질량 

\(L\) 

인덕턴스 

\(\displaystyle\frac{1}{k}\) 

역학적 컴플라이언스(compliance) 

\(C\) 

전기용량 

\(c\) 

감쇠 저항 

\(R\) 

저항 

\(F\) 

힘 

\(V\) 

전압 


다음의 회로는 교류 기전력 \(E_{0}\sin\omega t\)를 사용하는 직렬 RLC회로이다.

각 회로 소자에 걸리는 전압은$$V_{L}=L\frac{dI}{dt}=L\ddot{q},\,V_{R}=RI=R\dot{q},\,V_{C}=\frac{q}{C}$$이므로 키르히호프 법칙에 의해$$L\ddot{q}+R\dot{q}+\frac{q}{C}=E_{0}\sin\omega t$$이고, 이 회로에 흐르는 전류는$$I=-\frac{E_{0}}{\sqrt{R^{2}+\left(\omega L-\frac{1}{\omega C}\right)^{2}}}\sin(\omega t-\phi)$$이므로$$\begin{align*}V_{L}&=L\frac{dI}{dt}=-\frac{\omega LE_{0}}{\sqrt{R^{2}+\left(\omega L-\frac{1}{\omega C}\right)^{2}}}\cos(\omega t-\phi)\\&=V(\omega)\cos(\omega t-\phi)\end{align*}$$이다. \(V_{L}\)이 최대가 되게 하는 구동진동수(역학에서는 공명진동수) \(\omega_{\max}\)를 구하자.$$\frac{dV(\omega)}{d\omega}=\frac{\displaystyle LE_{0}\left(R^{2}-\frac{2L}{C}+\frac{2}{\omega^{2}C^{2}}\right)}{\displaystyle \left\{R^{2}+\left(\omega L-\frac{1}{\omega C}\right)^{2}\right\}^{\frac{3}{2}}}$$이므로$$\omega_{\max}=\frac{1}{\displaystyle\sqrt{LC-\frac{R^{2}C^{2}}{2}}}$$이다.

 

참고자료:

Analytical Mechanics 7th edition, Fowles, Cassiday, Cengage Learning

Classical Dynamics of Particles and Systems 5th edition, Thornton, Marion, Cengage Learning   

반응형
Posted by skywalker222